LSAT and Law School Admissions Forum

Get expert LSAT preparation and law school admissions advice from PowerScore Test Preparation.

 smile22
  • Posts: 135
  • Joined: Jan 05, 2014
|
#14307
I know that for this question we must strengthen the idea that the price of halibut will increase. For this question, I diagrammed a causal relationship:

decrease in supply :arrow: increase in price.

I incorrectly chose answer B. Can you please explain why B is incorrect? In the stimulus they note a relationship between supply and demand. Because there is a causal relationship between supply and price, is A correct because it is eliminating an alternate cause (a decrease in demand) that could lead to an increase in the price of halibut?
 Steve Stein
PowerScore Staff
  • PowerScore Staff
  • Posts: 1153
  • Joined: Apr 11, 2011
|
#14322
Hi Smile,

In that one, the author says that based on the laws of supply and demand, reducing the maximum catch of halibut will be likely to result in an increase in price.

The question asks for the choice that will most strengthen the author's argument. The problem with answer choice (B) is that given the laws of supply and demand, there is a connection between the supply and demand of most products, but this choice does not specify what the connection is or how strong.

Answer choice (A) provides the that demand for halibut will not decrease. If we add this assumption of at least constant demand (or perhaps even increasing demand), that would support the author's argument that lowering the halibut ceiling, perhaps reducing supply, would be likely to result in an increase in price.

I hope that's helpful--please let me know whether this is clear--thanks!

~Steve
 smile22
  • Posts: 135
  • Joined: Jan 05, 2014
|
#14329
Thank you for your reply!
 srcline@noctrl.edu
  • Posts: 243
  • Joined: Oct 16, 2015
|
#23254
Hello Steve

I've been reading this question for 10 minutes now and still not understanding it. I understand that this is a strengthen question and you're trying to find a gap. So the conclusion is " given the law of supple and demand these restrictions (lower ceilings) are likely to result in an increase in the price of fish.

I know C, D, E are irrelevant b/c we are not concerned with the production or other fish.

So is A saying that b/c there is a demand for halibut, people will not mind paying the price for it? hence: the demand for halibut will not decrease substantially after the new restrictions are imposed.

How does this imply that consumers would be hurt by the lower ceilings on halibut catches.Given that this is is a strengthen question how does answer choice A strengthen the relationship b/w supply and demand.?

Thankyou
Sarah
 Robert Carroll
PowerScore Staff
  • PowerScore Staff
  • Posts: 1787
  • Joined: Dec 06, 2013
|
#23302
Sarah,

Think about it this way - the law of supply and demand mentioned in the stimulus would result in an increased price if demand increased without supply increasing, or if supply decreased without demand decreasing. Thus, if demand decreases, then supply could potentially decrease by some amount before it would actually result in an increased price. Answer choice (A) allows us to include the fact that demand does not substantially decrease in the argument. If that decrease does not happen, then a supply decrease is more likely to cause a price increase. Because this is a Strengthen question, all we need to do is make the conclusion a little more likely to be true, and this answer choice does that. It does that by eliminating a possibility - what if the restrictions caused a supply decrease, but a substantial demand decrease also occurred? Then prices might not increase.

Robert Carroll
 Johnclem
  • Posts: 122
  • Joined: Dec 31, 2015
|
#28801
Hello,
I find this question a bit odd. Are we being asked to strengthen a causal premise ? ( unless I have the conclusion wrong )
Here is how I approached the question.

1- given the law of supply and demand these restrictions are likely to result in an increase in the price of fish ( restrictions --> increase in price of the fish )
C: consumers will be hurt by the ne lower ceilings on halibut catches .

Analysis :
Just because there is going to be a price increase doesn't mean consumers are going to be hurt . Maybe they won't care to purchase halibut.


A) correct : if there is still a demand for this fish, while there's a limit on its catch then it makes it true that prices will increase. ( supply and demand ).

B) wrong : so what if there is a connection ? Are the prices gonna increase or decrease or stay the same ?

C) wrong: even if the production is replaced doesn't mean the price of halibut would increase or decrease . So this is irrelevant.

D) wrong : we don't care about other fish. Only about halibut.

E) wrong : this weakens the argument , as if there is less demand not very likely that there will be an increase in price .

John
User avatar
 Jonathan Evans
PowerScore Staff
  • PowerScore Staff
  • Posts: 726
  • Joined: Jun 09, 2016
|
#28929
John,

It seems as though you may have glossed over the question stem here and just categorized this as a Strengthen question without paying close attention to the particular task. The question instructs you to find "which one of the following, if assumed, would do most to justify the claim that the price of halibut will increase?"

Therefore, irrespective of whether a price increase will hurt consumers, your job is to provide evidence that the new restrictions will in fact lead to higher prices. Even though the connection between high prices and hurt consumers is immaterial, the consumers are in fact important to your prephrase.

For example, you might prephrase that "demand for Halibut is not substantially lower right now than the amount being caught" or "given the new restrictions, demand for Halibut will exceed the new limited supply."

C, D, and E are terrible. B is weak garbage. Your analysis of the answer choices appears good. A is a clear match to what you might prephrase based on the question stem.
 MikeJones
  • Posts: 31
  • Joined: Oct 02, 2017
|
#40671
Jonathan Evans wrote:John,

It seems as though you may have glossed over the question stem here and just categorized this as a Strengthen question without paying close attention to the particular task. The question instructs you to find "which one of the following, if assumed, would do most to justify the claim that the price of halibut will increase?"

Therefore, irrespective of whether a price increase will hurt consumers, your job is to provide evidence that the new restrictions will in fact lead to higher prices. Even though the connection between high prices and hurt consumers is immaterial, the consumers are in fact important to your prephrase.

For example, you might prephrase that "demand for Halibut is not substantially lower right now than the amount being caught" or "given the new restrictions, demand for Halibut will exceed the new limited supply."

C, D, and E are terrible. B is weak garbage. Your analysis of the answer choices appears good. A is a clear match to what you might prephrase based on the question stem.
Hi Jonathan,

I realize this is a Strengthen question, but it occurred to be that both A and B appear to be required assumptions as well. Probably getting too far into the weeds here, but I just wanted to hear your thoughts on this. A clearly strengthens way more than B but it seems to me that B would be required as well if we were looking at these from a technical standpoint. Thanks!
 Claire Horan
PowerScore Staff
  • PowerScore Staff
  • Posts: 408
  • Joined: Apr 18, 2016
|
#40898
Hi Mike,

Even though the word "assumed" is used to direct you to take the answer choices as true, remember that this is a strengthen question rather than an assumption or justify the conclusion question. We know it is a strengthen question because it asks you which answer choice "would do most to justify the claim." In other words, which answer choice strengthens the claim the most?

If you rephrase the question stem that way, it is clear that answer choice (A) strengthens the claim that the price will increase. The law of supply and demand predicts that when supply goes down (due to ceilings on the catches) for the same amount of demand, the price increases. If supply and demand both decrease proportionally at the same time, the law of supply and demand does not predict a price increase. So knowing that demand will not decrease substantially strengthens the claim that the price of halibut will increase.

Answer choice (B), on the other hand, doesn't help strengthen the claim that the price will increase because it notes a connection but doesn't specify what the connection is (i.e. how the supply and demand for halibut are connected).

Thanks for the great question, and just be careful to identify the question type correctly!

Claire
 grunerlokka
  • Posts: 22
  • Joined: Jul 07, 2020
|
#81805
I understood what the 'gap' was in this stimuli's argument, but could you please tell me which of the two sentences is the conclusion. Isn't it the first one, 'consumers will be hurt...' ?

Get the most out of your LSAT Prep Plus subscription.

Analyze and track your performance with our Testing and Analytics Package.